Square root of matrix












2












$begingroup$


Let $A^2$ be a symmetric matrice such that the spectral theorem allows us to write
$$A^2 = Poperatorname{Diag}(lambda_1 dots lambda_n)P^T$$
Suppose $forall i in [![ 1, n]!], lambda_i geq 0$.



I want to know if the following proposition is true:
$$A^2 = Poperatorname{Diag}(lambda_1 dots lambda_n)P^T implies A = Poperatorname{Diag}(sqrt{lambda_1} dots sqrt{lambda_n})P^T$$










share|cite|improve this question











$endgroup$












  • $begingroup$
    yes, it is indeed true
    $endgroup$
    – pointguard0
    Jan 30 at 22:30










  • $begingroup$
    Please notice that my answer was initially only half-true; I presented the correct one now.
    $endgroup$
    – Blazej
    Jan 31 at 9:16
















2












$begingroup$


Let $A^2$ be a symmetric matrice such that the spectral theorem allows us to write
$$A^2 = Poperatorname{Diag}(lambda_1 dots lambda_n)P^T$$
Suppose $forall i in [![ 1, n]!], lambda_i geq 0$.



I want to know if the following proposition is true:
$$A^2 = Poperatorname{Diag}(lambda_1 dots lambda_n)P^T implies A = Poperatorname{Diag}(sqrt{lambda_1} dots sqrt{lambda_n})P^T$$










share|cite|improve this question











$endgroup$












  • $begingroup$
    yes, it is indeed true
    $endgroup$
    – pointguard0
    Jan 30 at 22:30










  • $begingroup$
    Please notice that my answer was initially only half-true; I presented the correct one now.
    $endgroup$
    – Blazej
    Jan 31 at 9:16














2












2








2


1



$begingroup$


Let $A^2$ be a symmetric matrice such that the spectral theorem allows us to write
$$A^2 = Poperatorname{Diag}(lambda_1 dots lambda_n)P^T$$
Suppose $forall i in [![ 1, n]!], lambda_i geq 0$.



I want to know if the following proposition is true:
$$A^2 = Poperatorname{Diag}(lambda_1 dots lambda_n)P^T implies A = Poperatorname{Diag}(sqrt{lambda_1} dots sqrt{lambda_n})P^T$$










share|cite|improve this question











$endgroup$




Let $A^2$ be a symmetric matrice such that the spectral theorem allows us to write
$$A^2 = Poperatorname{Diag}(lambda_1 dots lambda_n)P^T$$
Suppose $forall i in [![ 1, n]!], lambda_i geq 0$.



I want to know if the following proposition is true:
$$A^2 = Poperatorname{Diag}(lambda_1 dots lambda_n)P^T implies A = Poperatorname{Diag}(sqrt{lambda_1} dots sqrt{lambda_n})P^T$$







matrices diagonalization symmetric-matrices






share|cite|improve this question















share|cite|improve this question













share|cite|improve this question




share|cite|improve this question








edited Jan 30 at 22:44







Euler Pythagoras

















asked Jan 30 at 22:25









Euler PythagorasEuler Pythagoras

54212




54212












  • $begingroup$
    yes, it is indeed true
    $endgroup$
    – pointguard0
    Jan 30 at 22:30










  • $begingroup$
    Please notice that my answer was initially only half-true; I presented the correct one now.
    $endgroup$
    – Blazej
    Jan 31 at 9:16


















  • $begingroup$
    yes, it is indeed true
    $endgroup$
    – pointguard0
    Jan 30 at 22:30










  • $begingroup$
    Please notice that my answer was initially only half-true; I presented the correct one now.
    $endgroup$
    – Blazej
    Jan 31 at 9:16
















$begingroup$
yes, it is indeed true
$endgroup$
– pointguard0
Jan 30 at 22:30




$begingroup$
yes, it is indeed true
$endgroup$
– pointguard0
Jan 30 at 22:30












$begingroup$
Please notice that my answer was initially only half-true; I presented the correct one now.
$endgroup$
– Blazej
Jan 31 at 9:16




$begingroup$
Please notice that my answer was initially only half-true; I presented the correct one now.
$endgroup$
– Blazej
Jan 31 at 9:16










1 Answer
1






active

oldest

votes


















2












$begingroup$

This is partially wrong; see discussion below
This is false. If you know that $A^2= mathrm{diag}(x_1,...,x_n)$ in some basis, then in this basis $A=mathrm{diag}( pm sqrt{x_1},....,pm sqrt{x_n})$. Thus in general knowing $A^2$ there are $2^n$ choices for (symmetric) $A$. Of course taking square roots becomes unique if you restrict attention to positive matrices.



Correction It was pointed out to me in the comments that answer above is wrong without additional assumptions. Let's take a closer look. We know that $A$ commutes with $A^2$, so (since both are diagonalizable), there exists a common eigenbasis. If $A^2$ has nondegenerate spectrum then this already implies that $A$ is diagonal in the same basis as $A^2$. Otherwise we can group eigenvalues into blocks, so that (perhaps after reshuffling rows and columns) $A^2= mathrm{diag}(y_1,...,y_1,y_2,...,y_2,...,y_n,...y_k)$, where $y_i neq y_j$ for $i neq j$ and $y_i geq 0$ is repeated $g_i$ times. Now $A$ has to leave each eigenspace of $A^2$ invariant, so it is a direct sum of $g_i times g_i$ matrices of the form $sqrt{y_i} S_i$, where $S_i^2 = 1$, $S_i^T = S_i$. For $g_i=1$ the only matrices of this form are $pm 1$ (reproducing the previous answer), but in general this is not the only possibility. For example for $g_1 = 2$ there is the Pauli $x$ matrix $sigma_x = begin{pmatrix} 0 & 1 \ 1 & 0 end{pmatrix}$.






share|cite|improve this answer











$endgroup$













  • $begingroup$
    Your result works only if the $(x_i)$ are distinct; otherwise, even if $A$ is assumed to be symmetric, there are non-diagonal solutions in the form of orthogonal symmetries (up to a homothety).
    $endgroup$
    – loup blanc
    Jan 30 at 23:53










  • $begingroup$
    @loupblanc thank you, I corrected mu answer.
    $endgroup$
    – Blazej
    Jan 31 at 9:16












Your Answer





StackExchange.ifUsing("editor", function () {
return StackExchange.using("mathjaxEditing", function () {
StackExchange.MarkdownEditor.creationCallbacks.add(function (editor, postfix) {
StackExchange.mathjaxEditing.prepareWmdForMathJax(editor, postfix, [["$", "$"], ["\\(","\\)"]]);
});
});
}, "mathjax-editing");

StackExchange.ready(function() {
var channelOptions = {
tags: "".split(" "),
id: "69"
};
initTagRenderer("".split(" "), "".split(" "), channelOptions);

StackExchange.using("externalEditor", function() {
// Have to fire editor after snippets, if snippets enabled
if (StackExchange.settings.snippets.snippetsEnabled) {
StackExchange.using("snippets", function() {
createEditor();
});
}
else {
createEditor();
}
});

function createEditor() {
StackExchange.prepareEditor({
heartbeatType: 'answer',
autoActivateHeartbeat: false,
convertImagesToLinks: true,
noModals: true,
showLowRepImageUploadWarning: true,
reputationToPostImages: 10,
bindNavPrevention: true,
postfix: "",
imageUploader: {
brandingHtml: "Powered by u003ca class="icon-imgur-white" href="https://imgur.com/"u003eu003c/au003e",
contentPolicyHtml: "User contributions licensed under u003ca href="https://creativecommons.org/licenses/by-sa/3.0/"u003ecc by-sa 3.0 with attribution requiredu003c/au003e u003ca href="https://stackoverflow.com/legal/content-policy"u003e(content policy)u003c/au003e",
allowUrls: true
},
noCode: true, onDemand: true,
discardSelector: ".discard-answer"
,immediatelyShowMarkdownHelp:true
});


}
});














draft saved

draft discarded


















StackExchange.ready(
function () {
StackExchange.openid.initPostLogin('.new-post-login', 'https%3a%2f%2fmath.stackexchange.com%2fquestions%2f3094199%2fsquare-root-of-matrix%23new-answer', 'question_page');
}
);

Post as a guest















Required, but never shown

























1 Answer
1






active

oldest

votes








1 Answer
1






active

oldest

votes









active

oldest

votes






active

oldest

votes









2












$begingroup$

This is partially wrong; see discussion below
This is false. If you know that $A^2= mathrm{diag}(x_1,...,x_n)$ in some basis, then in this basis $A=mathrm{diag}( pm sqrt{x_1},....,pm sqrt{x_n})$. Thus in general knowing $A^2$ there are $2^n$ choices for (symmetric) $A$. Of course taking square roots becomes unique if you restrict attention to positive matrices.



Correction It was pointed out to me in the comments that answer above is wrong without additional assumptions. Let's take a closer look. We know that $A$ commutes with $A^2$, so (since both are diagonalizable), there exists a common eigenbasis. If $A^2$ has nondegenerate spectrum then this already implies that $A$ is diagonal in the same basis as $A^2$. Otherwise we can group eigenvalues into blocks, so that (perhaps after reshuffling rows and columns) $A^2= mathrm{diag}(y_1,...,y_1,y_2,...,y_2,...,y_n,...y_k)$, where $y_i neq y_j$ for $i neq j$ and $y_i geq 0$ is repeated $g_i$ times. Now $A$ has to leave each eigenspace of $A^2$ invariant, so it is a direct sum of $g_i times g_i$ matrices of the form $sqrt{y_i} S_i$, where $S_i^2 = 1$, $S_i^T = S_i$. For $g_i=1$ the only matrices of this form are $pm 1$ (reproducing the previous answer), but in general this is not the only possibility. For example for $g_1 = 2$ there is the Pauli $x$ matrix $sigma_x = begin{pmatrix} 0 & 1 \ 1 & 0 end{pmatrix}$.






share|cite|improve this answer











$endgroup$













  • $begingroup$
    Your result works only if the $(x_i)$ are distinct; otherwise, even if $A$ is assumed to be symmetric, there are non-diagonal solutions in the form of orthogonal symmetries (up to a homothety).
    $endgroup$
    – loup blanc
    Jan 30 at 23:53










  • $begingroup$
    @loupblanc thank you, I corrected mu answer.
    $endgroup$
    – Blazej
    Jan 31 at 9:16
















2












$begingroup$

This is partially wrong; see discussion below
This is false. If you know that $A^2= mathrm{diag}(x_1,...,x_n)$ in some basis, then in this basis $A=mathrm{diag}( pm sqrt{x_1},....,pm sqrt{x_n})$. Thus in general knowing $A^2$ there are $2^n$ choices for (symmetric) $A$. Of course taking square roots becomes unique if you restrict attention to positive matrices.



Correction It was pointed out to me in the comments that answer above is wrong without additional assumptions. Let's take a closer look. We know that $A$ commutes with $A^2$, so (since both are diagonalizable), there exists a common eigenbasis. If $A^2$ has nondegenerate spectrum then this already implies that $A$ is diagonal in the same basis as $A^2$. Otherwise we can group eigenvalues into blocks, so that (perhaps after reshuffling rows and columns) $A^2= mathrm{diag}(y_1,...,y_1,y_2,...,y_2,...,y_n,...y_k)$, where $y_i neq y_j$ for $i neq j$ and $y_i geq 0$ is repeated $g_i$ times. Now $A$ has to leave each eigenspace of $A^2$ invariant, so it is a direct sum of $g_i times g_i$ matrices of the form $sqrt{y_i} S_i$, where $S_i^2 = 1$, $S_i^T = S_i$. For $g_i=1$ the only matrices of this form are $pm 1$ (reproducing the previous answer), but in general this is not the only possibility. For example for $g_1 = 2$ there is the Pauli $x$ matrix $sigma_x = begin{pmatrix} 0 & 1 \ 1 & 0 end{pmatrix}$.






share|cite|improve this answer











$endgroup$













  • $begingroup$
    Your result works only if the $(x_i)$ are distinct; otherwise, even if $A$ is assumed to be symmetric, there are non-diagonal solutions in the form of orthogonal symmetries (up to a homothety).
    $endgroup$
    – loup blanc
    Jan 30 at 23:53










  • $begingroup$
    @loupblanc thank you, I corrected mu answer.
    $endgroup$
    – Blazej
    Jan 31 at 9:16














2












2








2





$begingroup$

This is partially wrong; see discussion below
This is false. If you know that $A^2= mathrm{diag}(x_1,...,x_n)$ in some basis, then in this basis $A=mathrm{diag}( pm sqrt{x_1},....,pm sqrt{x_n})$. Thus in general knowing $A^2$ there are $2^n$ choices for (symmetric) $A$. Of course taking square roots becomes unique if you restrict attention to positive matrices.



Correction It was pointed out to me in the comments that answer above is wrong without additional assumptions. Let's take a closer look. We know that $A$ commutes with $A^2$, so (since both are diagonalizable), there exists a common eigenbasis. If $A^2$ has nondegenerate spectrum then this already implies that $A$ is diagonal in the same basis as $A^2$. Otherwise we can group eigenvalues into blocks, so that (perhaps after reshuffling rows and columns) $A^2= mathrm{diag}(y_1,...,y_1,y_2,...,y_2,...,y_n,...y_k)$, where $y_i neq y_j$ for $i neq j$ and $y_i geq 0$ is repeated $g_i$ times. Now $A$ has to leave each eigenspace of $A^2$ invariant, so it is a direct sum of $g_i times g_i$ matrices of the form $sqrt{y_i} S_i$, where $S_i^2 = 1$, $S_i^T = S_i$. For $g_i=1$ the only matrices of this form are $pm 1$ (reproducing the previous answer), but in general this is not the only possibility. For example for $g_1 = 2$ there is the Pauli $x$ matrix $sigma_x = begin{pmatrix} 0 & 1 \ 1 & 0 end{pmatrix}$.






share|cite|improve this answer











$endgroup$



This is partially wrong; see discussion below
This is false. If you know that $A^2= mathrm{diag}(x_1,...,x_n)$ in some basis, then in this basis $A=mathrm{diag}( pm sqrt{x_1},....,pm sqrt{x_n})$. Thus in general knowing $A^2$ there are $2^n$ choices for (symmetric) $A$. Of course taking square roots becomes unique if you restrict attention to positive matrices.



Correction It was pointed out to me in the comments that answer above is wrong without additional assumptions. Let's take a closer look. We know that $A$ commutes with $A^2$, so (since both are diagonalizable), there exists a common eigenbasis. If $A^2$ has nondegenerate spectrum then this already implies that $A$ is diagonal in the same basis as $A^2$. Otherwise we can group eigenvalues into blocks, so that (perhaps after reshuffling rows and columns) $A^2= mathrm{diag}(y_1,...,y_1,y_2,...,y_2,...,y_n,...y_k)$, where $y_i neq y_j$ for $i neq j$ and $y_i geq 0$ is repeated $g_i$ times. Now $A$ has to leave each eigenspace of $A^2$ invariant, so it is a direct sum of $g_i times g_i$ matrices of the form $sqrt{y_i} S_i$, where $S_i^2 = 1$, $S_i^T = S_i$. For $g_i=1$ the only matrices of this form are $pm 1$ (reproducing the previous answer), but in general this is not the only possibility. For example for $g_1 = 2$ there is the Pauli $x$ matrix $sigma_x = begin{pmatrix} 0 & 1 \ 1 & 0 end{pmatrix}$.







share|cite|improve this answer














share|cite|improve this answer



share|cite|improve this answer








edited Jan 31 at 9:15

























answered Jan 30 at 22:33









BlazejBlazej

1,632620




1,632620












  • $begingroup$
    Your result works only if the $(x_i)$ are distinct; otherwise, even if $A$ is assumed to be symmetric, there are non-diagonal solutions in the form of orthogonal symmetries (up to a homothety).
    $endgroup$
    – loup blanc
    Jan 30 at 23:53










  • $begingroup$
    @loupblanc thank you, I corrected mu answer.
    $endgroup$
    – Blazej
    Jan 31 at 9:16


















  • $begingroup$
    Your result works only if the $(x_i)$ are distinct; otherwise, even if $A$ is assumed to be symmetric, there are non-diagonal solutions in the form of orthogonal symmetries (up to a homothety).
    $endgroup$
    – loup blanc
    Jan 30 at 23:53










  • $begingroup$
    @loupblanc thank you, I corrected mu answer.
    $endgroup$
    – Blazej
    Jan 31 at 9:16
















$begingroup$
Your result works only if the $(x_i)$ are distinct; otherwise, even if $A$ is assumed to be symmetric, there are non-diagonal solutions in the form of orthogonal symmetries (up to a homothety).
$endgroup$
– loup blanc
Jan 30 at 23:53




$begingroup$
Your result works only if the $(x_i)$ are distinct; otherwise, even if $A$ is assumed to be symmetric, there are non-diagonal solutions in the form of orthogonal symmetries (up to a homothety).
$endgroup$
– loup blanc
Jan 30 at 23:53












$begingroup$
@loupblanc thank you, I corrected mu answer.
$endgroup$
– Blazej
Jan 31 at 9:16




$begingroup$
@loupblanc thank you, I corrected mu answer.
$endgroup$
– Blazej
Jan 31 at 9:16


















draft saved

draft discarded




















































Thanks for contributing an answer to Mathematics Stack Exchange!


  • Please be sure to answer the question. Provide details and share your research!

But avoid



  • Asking for help, clarification, or responding to other answers.

  • Making statements based on opinion; back them up with references or personal experience.


Use MathJax to format equations. MathJax reference.


To learn more, see our tips on writing great answers.




draft saved


draft discarded














StackExchange.ready(
function () {
StackExchange.openid.initPostLogin('.new-post-login', 'https%3a%2f%2fmath.stackexchange.com%2fquestions%2f3094199%2fsquare-root-of-matrix%23new-answer', 'question_page');
}
);

Post as a guest















Required, but never shown





















































Required, but never shown














Required, but never shown












Required, but never shown







Required, but never shown

































Required, but never shown














Required, but never shown












Required, but never shown







Required, but never shown







Popular posts from this blog

Can a sorcerer learn a 5th-level spell early by creating spell slots using the Font of Magic feature?

Does disintegrating a polymorphed enemy still kill it after the 2018 errata?

A Topological Invariant for $pi_3(U(n))$